Categories
Exam Questions Harvard Leontief Undergraduate

Harvard. Undergraduate mathematical economics. Schumpeter, Leontief, Goodwin. 1933-1950

 

 

Joseph Schumpeter introduced a one semester undergraduate course “Introduction to the Mathematical Treatment of Economic Theory” in the first semester of the 1933-34 academic year at Harvard. Schumpeter taught the course three times and it was taught from 1935-36 through 1947-48 by Wassily Leontief. The course was then continued by Richard Goodwin in 1949-50. This post presents a grab-bag of information that includes early and a late course description, annual enrollment data, a course outline from 1945-46 and five exams. Links to all earlier posts for the course available at Economics in the Rear-view Mirror have been included as well.

Some of the backstory to this course is included in this earlier post (memo by Crum of 4 April 1933 and a list of topics to be covered).

_______________

Course Announcement, 1933-34

Economics 8a 1hfIntroduction to the Mathematical Treatment of Economic Theory

Half-course (first half-year). Mon., 4 to 6, and a third hour at the pleasure of the instructor. Professor Schumpeter, and other members of the Department.

Economics 8a is open to those who have passed Economics A, and Mathematics A, or its equivalent. The aim of this course is to acquaint such students as may wish it with the elements of the mathematical technique necessary to understand the simpler contributions to the mathematical theory of economics.

Source:  Announcement of the Courses of Instruction offered by the Faculty of Arts and Sciences 1933-34 (Second edition) in Official Register of Harvard University, Vol. XXX, No. 39 (September 20, 1933), p. 126.

*  *  *  *  *  *  *

Course Enrollment, 1933-34

[Economics] 8a 1hf. Professor Schumpeter. — Introduction to the Mathematical Treatment of Economic Theory.

15 Graduates, 3 Seniors, 5 Others. Total 23.

Source: Harvard University. Report of the President of Harvard College and Reports of Departments for 1933-1934, p. 85.

*  *  *  *  *  *  *

Exam not found for Economics 8a, 1933-34

_______________

Course Enrollment, 1934-35

[Economics] 8a 1hf. Professor Schumpeter. — Introduction to the Mathematical Treatment of Economic Theory.

2 Seniors, 1 Junior, 1 Sophomore. Total 4.

Source: Harvard University. Report of the President of Harvard College and Reports of Departments for 1934-1935, p. 81.

*  *  *  *  *  *  *

 1935 final exam questions.

_______________

Course Enrollment, 1935-36

[Economics] 8a 2hf. Asst. Professor Leontief. — Introduction to the Mathematical Treatment of Economic Theory.

4 Juniors, 2 Sophomores. Total 6.

Source: Harvard University. Report of the President of Harvard College and Reports of Departments for 1935-1936, p. 82.

*  *  *  *  *  *  *

Implicit course outline and course readings with the 1936 exam questions.

_______________

Course Enrollment, 1936-37

[Economics] 4a 2hf. Asst. Professor Leontief. — Introduction to the Mathematical Treatment of Economic Theory.

1 Graduate, 2 Seniors, 3 Juniors, 2 Sophomores, 1 Other. Total 9.

Source: Harvard University. Report of the President of Harvard College and Reports of Departments for 1936-1937, p. 92.

*  *  *  *  *  *  *

Final Examination, 1936-37
HARVARD UNIVERSITY
ECONOMICS 4a

Answer at least THREE questions: one in each group

Group I

  1. Discuss the relation between the production function of an enterprise and its cost curve.

 

Group II

  1. Given a cost of a single plant:
    C=\frac{1}{A+X}+BX
    where indicates the total cost, the total output, and the magnitudes of the two constants are such
    that A< 0 and B> 1/A.
    Derive the total cost curve of an enterprise which consists of two identical plants of this kind.
  2. A monopolist sells in two markets a commodity produced without costs. The total revenue, R1, obtained from the sale of qunits of this commodity in the first market is given by:
    {{R}_{1}}=A{{q}_{1}}+Bq_{1}^{2}\text{ }\left( A>0,\text{ }B<\text{ }0 \right)
    The sale of qunits in the second market nets:
    {{R}_{2}}=K{{q}_{2}}+Lq_{2}^{2}\text{ }\left( K>0,\text{ }L<\text{ }0 \right)
    Compute the prices which this monopolist would charge (a) with discrimination between the two markets; (b) without discrimination.

 

Group III

  1. Prove that marginal costs are increasing in the point of minimum average costs.
  2. Prove that a tax on profits cannot affect the output of an enterprise unless it induces it to suspend its operations.

 

Source: Harvard University Archives. Examination Papers. Finals 1937. (HUC 7000.28) Vol. 79. Faculty of Arts and Sciences. Papers Printed for Final Examinations. History, History of Religions, …, Economics, …, Military Science, Naval Science. January-June, 1937.

_______________

Course Enrollment, 1937-38

[Economics] 4a 2hf. Asst. Professor Leontief. — Introduction to the Mathematical Treatment of Economic Theory.

2 Graduates, 2 Seniors, 6 Juniors, 1 Sophomore. Total 11.

Source: Harvard University. Report of the President of Harvard College and Reports of Departments for 1937-1938, p. 85.

*  *  *  *  *  *  *

Final Examination, 1937-38
HARVARD UNIVERSITY
ECONOMICS 4a2

Answer THREE questions including question 1. Devote to discussion of question 1 about one hour and a half.

  1. Discuss fully the relation between the production function and the cost curve of an enterprise.
  2. Given:
    1. The cost curve of a monopolist:
      C= A+ BQ+ CQ2
      C indicates the total cost, the total output, A, B, C,are given constants.
    2. The demand function for his product in Market I.
      q1= a1b1p1
      qis the quantity consumed for his product in Market I at the price p1.
      a1and bare given constants
    3. The demand function for his product in Market II.
      q2= a2b2p2
      q2is the quantity taken in at the price p2;
      aand bare given constants.
      The monopolist is able to discriminate between the two markets provided the difference between the two prices is not larger than K
      Find (and express in terms of the given constants) that the value of Kwhich would maximize the sales qin the first market.
  3. Given:
    1. A, monopolist’s cost curve:
      C = A+ BQ+ CQ
    2. The demand curve for his product:
      p= a bQ
      stands for total costs, Q for total output, for the market price, A, B, C, d, and are constants.
      A subsidy at dollars is paid to the monopolist per unit of output.
      Find how large the subsidy must be in order to induce him to produce and sell twice as much as he would without the subsidy.
  4. Is it possible that the average costs of an enterprise are increasing with the output while the marginal costs are decreasing at the same time?
    Give and answer and demonstrate that it is correct.

 

Source: Harvard University Archives. Harvard University Final Examinations, 1853-2001. (HUC 7000.28) Box 4. Faculty of Arts and Sciences. Papers Printed for Final Examinations. History, History of Religions, …, Economics, …, Military Science, Naval Science. January-June, 1938.

_______________

Course Enrollment, 1938-39

[Economics] 4a 2hf. Asst. Professor Leontief. — Introduction to the Mathematical Treatment of Economic Theory.

2 Graduates, 2 Seniors, 2 Juniors, 1 Sophomore. Total 7.

Source: Harvard University. Report of the President of Harvard College and Reports of Departments for 1938-1939, pp. 97-98.

*  *  *  *  *  *  *

Exam not found for Economics 4a, 1938-39

_______________

Course Enrollment, 1939-40

[Economics] 4a 2hf. Associate Professor Leontief. — Introduction to the Mathematical Treatment of Economic Theory.

1 Graduate, 1 Sophomore. Total 5.

Source: Harvard University. Report of the President of Harvard College and Reports of Departments for 1939-1940, p. 98.

*  *  *  *  *  *  *

Final Examination, 1939-40
HARVARD UNIVERSITY
ECONOMICS 4a2

Answer four questions including question 1.

  1. Discuss the relation between the marginal costs of an enterprise and the marginal productivities of the factors used in production.
  2. An enterprise manufactures two commodities X and Y, using two factors of production, V and W. The production function is x(yb– 1) = vnwm.
    Given the prices px, py, pvand pwwrite down the equations which determine the most profitable outputs of X and Y and the corresponding inputs of V and W.
  3. Given:
    1. The total cost curve of a monopolist
      C = A + Kxand
    2. the market demand curve for his product
      p = B – Lx,
      p is the price and x the quantity of the commodity produced and sold. A, K, B and L are positive constants.
      An excise tax of z dollars per unit of output is being levied.
      What magnitude of z (expressed in terms of the given constants) would maximize the total tax receipts?
  4. Prove that the price of labor will exceed its marginal value productivity if
    1. labor is the only factor of production used in manufacture of a given commodity,
    2. the producer of this commodity sells his output on a purely competitive market, but is the only (“monopsonistic”) buyer of the particular kind of labor used in his plant,
    3. The supply curve of labor is negatively inclined.
  5. Discuss the problem of price discrimination by a monopolist.

 

Source: Harvard University Archives. Harvard University Final Examinations, 1853-2001. (HUC 7000.28) Box 5. Faculty of Arts and Sciences. Papers Printed for Final Examinations. History, History of Religions, …, Economics, …, Military Science, Naval Science. June, 1940.

_______________

Economics 4a not offered in 1940-41

_______________

Course Enrollment, 1941-42

[Economics] 4a 2hf. Associate Professor Leontief. — Introduction to the Mathematical Treatment of Economic Theory.

1 Graduate, 5 Seniors, 8 Juniors, 3 Sophomores, 1 Freshman. Total 18.

Source: Harvard University. Report of the President of Harvard College and Reports of Departments for 1941-1942, p. 62.

*  *  *  *  *  *  *

Course Outline Economics 4a 1941-42 (and 1942-43)

https://www.irwincollier.com/harvard-intro-to-mathematical-economics-schumpeter-leontief-1935-42/

*  *  *  *  *  *  *

Final Examination, 1941-42
HARVARD UNIVERSITY
ECONOMICS 4a

Answer one question in each of the following three groups:

(a) 1 or 2
(b) 3 or 4
(c) 5 or 6

  1. Describe in detail the relation between a production function and the corresponding cost function.
  2. Show that the slope of a supply curve of a single enterprise is positive.
  3. Show that a total cost curve can be of such a shape that the marginal costs are increasing but the average costs decreasing throughout its whole length. Give example.
  4. The cost curve of an enterprise is
    C = A + x + Bx2+ Kx3
    (C are the total costs, x – the output, A, B, and K – constants).
    What is the lowest competitive price at which the owner will find it profitable to operate the plant rather than to cease production entirely?
  5. An enterprise consists of two identical plants. Each has a following cost curve:
    C = A + Bx2+ x3
    (C are the total costs, x – the output, A and B are constants).
    Compute the combined cost curve of the whole enterprise.
  6. Given a production function y = f(x,z)
    (y is the amount of product, p– its price, x and z inputs of two factors, pand p– their respective prices.)
    The producer maximizes his profits under conditions of pure competition. Show that an increase of the price pof factor x will reduce the amount (x) of this factor used in the process of production.

 

Source: Harvard University Archives. Harvard University Final Examinations, 1853-2001. (HUC 7000.28) Box 6. Faculty of Arts and Sciences. Papers Printed for Final Examinations. History, History of Religions, …, Economics, …, Military Science, Naval Science. June, 1942.

_______________

Course Description, 1942-43

Economics 4a 1hfIntroduction to the Mathematical Treatment of Economic Theory. Half-course (first half-year). Mon.4 to 6. Associate Professor Leontief.

Economics A and Mathematics A, or their equivalents, are prerequisites for this course.
The course is intended to instruct beginners in economic theory (having had elementary mathematical training) in the application of elementary mathematical methods in economics and at the same time to enable them to understand some of the major contributions to economic theory made by such writers as Marshall, Cournot, Walras, and Edgeworth.

Source:  Official Register of Harvard University, Vol. XXXIX, No. 45 (June 30, 1942). Division of History, Government, and Economics Containing an Announcement for 1942-43. 

*  *  *  *  *  *  *

Course Enrollment, 1942-43

[Economics] 4a 1hf. Associate Professor Leontief. — Introduction to the Mathematical Treatment of Economic Theory.

1 Graduate, 2 Seniors, 4 Juniors, 2 Sophomores, 1 Public Administration. Total 10.

Source: Harvard University. Report of the President of Harvard College and Reports of Departments for 1942-1943, p. 46.

*  *  *  *  *  *  *

Exam not found for Economics 4a, 1942-43

_______________

Course Enrollment, 1943-44

[Economics] 4a. (winter term) Associate Professor Leontief. — Introduction to the Mathematical Treatment of Economic Theory.

2 Juniors in ROTC, 1 Radcliffe, 3 Seniors, 4 Navy (V-12). Total 10.

Source: Harvard University. Report of the President of Harvard College and Reports of Departments for 1943-1944, p. 56.

*  *  *  *  *  *  *

Exam not found for Economics 4a, 1943-44

_______________

Economics 4a not offered in 1944-45

_______________

Course Enrollment, 1945-46

[Economics] 4a. (fall term) Associate Professor Leontief. — Introduction to the Mathematical Treatment of Economic Theory.

1 Senior, 2 Juniors, 3 Sophomores, 2 Radcliffe. Total 8.

Source: Harvard University. Report of the President of Harvard College and Reports of Departments for 1945-1946, p. 58.

*  *  *  *  *  *  *

Course Outline, 1945-46

INTRODUCTION TO THE MATHEMATICAL TREATMENT OF ECONOMIC THEORY
Economics 4a
1945-46, Fall Term

  1. Introductory remarks.
    Profit function.
    Maximizing profits.
  2. Cost functions: Total costs, fixed costs, variable costs, average costs, marginal costs, increasing and decreasing marginal costs.
    Minimizing average total and average variable costs.
  3. Revenue function.
    Price and marginal revenue.
    Demand function
    Elasticity and flexibility.
  4. Maximizing the net revenue (profits).
    Monopolistic maximum.
    Competitive maximum.
    Supply function.
  5. Joint costs and accounting methods of cost imputation.
    Multiple plants.
    Price discrimination.
  6. Production function.
    Marginal productivity.
    Increasing and decreasing productivity.
    Homogeneous and non-homogeneous production functions.
  7. Maximizing net revenue, second method.
    Minimizing costs for a fixed output.
    Marginal costs and marginal productivity.
  8. Introduction into the theory of consumers’ behavior.
    Indifference curves and the utility function.
  9. Introduction to the theory of the market.
    Concept of market equilibrium.
    Duopoly, bilateral monopoly.
    Pure competition.
    Monopoly.
  10. Time lag and time sequences.
  11. Introduction into the theory of general equilibrium.

 

Reading: R. G. D. Allen, Mathematical Analysis for Economists.

Evans, Introduction into Mathematical Economics.

Antoine Cournot, Researches into the Mathematical Principles of the Theory of Wealth.

Jacob L. Mosak, General Equilibrium Theory in International Trade.

Weekly problems.

Source: Harvard University Archives. Syllabi, course outlines and reading lists in Economics, 1895-2003. HUC 8522.2.1, Box 3, Folders “1945-1946 (1 of 2)”.

*  *  *  *  *  *  *

Final Examination, 1945-46
1945-46
HARVARD UNIVERSITY
ECONOMICS 4a
Introduction to Mathematical Economics

Answer any three questions.

  1. Show the relationship between the total cost curve and the supply curve of an enterprise.
  2. Show that, at the point of optimum output, the marginal costs of an enterprise are equal to the price of any cost factor divided by its marginal productivity.
  3. A consumer has an income of qdollars in the first and of ydollars in the second year. Although the combined expenditures in the two years equal y1+ y2he can spend more than yin the first year, and correspondingly less in the second year or vice versa. In both years, he purchases one kind of consumers’ goods, its price being pdollars in the first and pdollars per unit in the second year. The utility function which the consumer maximizes is u= f(x1, x2) where is the utility level, xand xthe quantities consumed in the first and second year respectively.
    1. Derive the equations which determine the optimum magnitudes of xand x2.
    2. Show that an increase of the price p1, with p2, y1,yremaining constant, might increase x1.
  4. The demand, q, for the product of a monopolist depends upon the price, p, of his produce and the amount of money, y, which he spends on advertising. The total production cost, c, depends upon the quantity of output, q. Given the demand function: q=\frac{A}{p}+{{y}^{{1}/{4}\;}}-p
    and the total (production) cost function = q
    where is a positive constant;
    Determine the output, the price, and the advertising outlay which would maximize the profits (total revenue minus total outlay) of this enterprise.
  5. The well-being, u, of a worker depends upon the amount, x, of consumers’ goods which he can buy with his daily wage, and the number of hours of leisure, y, which remain to him after he finishes his daily work:
    u= f(x, y)

    1. Derive the equations determining the number of hours (call it l) of daily work which he will be willing to do at the wage of dollars per hour, if the price of the consumers’ good is dollars per unit.
    2. Show that an increase of the hourly wage rate might reduce the number of hours which the worker will choose to work.

 

Source: Harvard University Archives. Harvard University Final Examinations, 1853-2001. (HUC 7000.28) Box 11. Faculty of Arts and Sciences. Papers Printed for Final Examinations. History, History of Religions, …, Economics, …, Military Science, Naval Science. January, 1946.

_______________

Economics 4a not offered in 1946-47

_______________

Course Enrollment, 1947-48

[Economics] 4a. Professor Leontief. — Introduction to the Mathematical Treatment of Economic Theory (Sp).

2 Graduates, 6 Seniors, 8 Juniors, 1 Sophomore, 2 Public Administration, 1 Radcliffe. Total 20.

Source: Harvard University. Report of the President of Harvard College and Reports of Departments for 1947-1948, p. 89.

*  *  *  *  *  *  *

Reading list and midterm and final examination question, 1947-48

_______________

Economics 4a not offered in 1948-49

_______________

Course Enrollment, 1949-50

[Economics] 104 (formerly Economics 4a). Assistant Professor Goodwin. — Introduction to the Mathematical Treatment of Economic Theory (Sp).

3 Graduates, 6 Seniors, 1 Junior, 2 Sophomores, 1 Public Administration, 1 Radcliffe. Total 14.

Source: Harvard University. Report of the President of Harvard College and Reports of Departments for 1949-1950, p.72.

*  *  *  *  *  *  *

Course Texts on Library Reserve, 1945-46

R.G.D. Allen. Mathematical analysis for economists

W.L. Crum. Rudimentary mathematics for economists and statisticians

P.A. Samuelson. Foundations of economic analysis.

Source: Harvard University Archives. Syllabi, course outlines and reading lists in Economics, 1895-2003. HUC 8522.2.1, Box 4, Folders “1949-1950 (1 of 3)”.

_______________

Image Sources: Schumpeter and Leontief from Harvard Class Album 1950, Goodwin from Harvard Class Album 1951.

Categories
Harvard Suggested Reading Syllabus

Harvard. Advanced Economic Theory, Second Term. Schumpeter, 1948

 

 

In 1947-48 the advanced economic theory sequence of two semester courses featured the pairing of Gottfried Haberler and Joseph Schumpeter in the Winter and Spring terms, respectively. In this post you will find course enrollment data along with the course outline and assigned readings for the Spring term taught by Schumpeter. Alas I could not find the final examination questions for this course in the otherwise fairly complete collection of course examinations in the Harvard Archives.

Materials from the Winter Term course 1947-48 taught by Gottfried Haberler.

__________________

Course Enrollment

[Economics] 103b. Professor Schumpeter.—Advanced Economic Theory (Sp).

Total 10:  8 Graduates, 1 Public Administration, 1 Radcliffe.

Source: Harvard University. Report of the President of Harvard College, 1947-48, p. 90.

__________________

Advanced Economic Theory
Joseph Schumpeter

Economics 103b
Spring Term 1947-48

Plan of Course and Suggestions for Reading

The plan of the course is to start from and to build upon Professor Haberler’s lectures in the Fall Term (103a). We shall start from the statics of equilibrium and then discuss at some lengths the use and limitations of the method of Comparative Statics. After this, we shall survey various Dynamic Models. These models will be made the starting points of excursions into relevant fields of pure and applied theory.

Professor Haberler’s reading list remains in force. Wicksell’s Lectures I being particularly recommended. In addition, perusal of the following items will prove helpful. The more important ones are marked by an asterisk. The list is intended to cover also suggestions for the reading period.

J. Tinbergen*, Suggestions on Quantitative Business Cycle Theory, Econometrica, July 1935.

F. Modigliani*, Liquidity Preference, Interest, and Money, Econometrica, January 1944.

N. Kaldor*, Stability and Full Employment, Economic Journal, December 1938

F. Lavington, Approach to a Theory of Business Risks, Economic Journal, June 1925

L. Metzler, Factors Governing the Length of Inventory Cycles, Review of Economic Statistics, February 1947

M. V. Jones, Secular and Cyclical Saving Propensities, Journal of Business of the University of Chicago, January 1944

L. M. Lachmann, Uncertainty and Liquidity Preference, Economica, August 1937

M. Kalecki, A Theorem on Technical Progress, Review of Economic Studies, June 1941

P. A. Samuelson*, Foundations of Economic Analysis, 1947

 

Source: Harvard University Archives. Syllabi, course outlines and reading lists in economics, 1895-2003. Box 4. Folder “Economics, 1947-48 (2 of 2)”.

Image Source: Joseph Schumpeter in Harvard Class Album 1946.

 

 

Categories
Exam Questions Harvard Socialism Suggested Reading Syllabus

Harvard. Economics of socialism. Outline, readings, final exam. Schumpeter, 1943-44

 

Earlier Economics in the Rear-view Mirror posted the course outline and final examination for Joseph Schumpeter’s course on the economics of socialism that was given in the second semester of 1945-46. None of the final examination questions were shared between these two years, so together the exams provide a better idea of what was actually covered than either alone.

______________________

Course Announcement

Economics 11b. Economics of Socialism

Half-course (winter term). Mon., Wed., and (at the pleasure of the instructor) Fri., at  10. Professor Schumpeter.

 

Source: Announcement of the Courses of Instruction Offered by the Faculty of Arts and Sciences During 1943-44. Official Register of Harvard University, Vol. 40, No. 21 (September 29, 1943), p. 33.

______________________

Course Enrollment

[Economics] 11b (winter term) Professor Schumpeter. –Economics of Socialism.

Total 26:  3 Graduates, 5 Seniors, 2 Juniors, 1 Sophomore, 13 Navy.

 

Source: Harvard University. Report of the President of Harvard College and Reports of Departments for 1943-44, p. 56.

______________________

ECONOMICS 11b
1943-44
OUTLINE AND ASSIGNMENTS

  1. FIRST TWO WEEKS: The Socialist Issue.

Socialist ideas and socialist parties. Socialism and the labor movement. Laborite and intellectualist socialism. The Definition of Socialism.

H. W. Laidler*, History of Socialist Thought, 1927.
T.M. Sogge, “Industrial Classes in the U. S.  in 1930,” Journal of the American Statistical Association, vol. 28 (1933), pp. 199-203.
Encyclopaedia of the Social Sciences, article on Socialist and Labor Parties.

  1. THIRD TO FIFTH WEEK: The Theory of Centralist Socialism.

O. Lange and F. M. Taylor*, The Economic Theory of Socialism.
H. D. Dickinson, Economics of Socialism, 1939.

  1. SIXTH TO NINTH WEEK: The Economic Interpretation of History. The Class Struggle, and the Marxist Theory of Capitalism.

Karl Marx, Capital, Volume I, chs. I, IV, V, VI.
Marx and Engels, The Communist Manifesto
Paul M. Sweezy*, The Theory of Capitalist Development, 1942, chs. I-VI (pp. 1-108).

  1. TENTH TO TWELFTH WEEK: The Socialist Theory of the State and of the Proletarian Revolution, Imperialism, National Socialism.

V.I. Lenin, State and Revolution.
V. I. Lenin, Imperialism.
M. Dobb, Political Economy and Capitalism, ch. VII.
Paul M. Sweezy*, The Theory of Capitalist Development, Chs. XIII-XIX.

READING PERIOD ASSIGNMENT

Read E. Bernstein, Evolutionary Socialism, especially pp. 18-95, and survey again the items in the reading list marked *.

 

Source:Harvard University Archives. Syllabi, course outlines and reading lists in economics, 1895-2003 (HUC 8522.2.1). Box 3, Folder “Economics, 1943-1944 (2 of 2)”.

______________________

1943-44
HARVARD UNIVERSITY
ECONOMICS 11b
[Final. February, 1944]

One question may be omitted. Arrange your answers in the order of the questions.

  1. Describe briefly the emergence of either the English Independent Labour Party or the German Social Democratic Party.
  2. In the Second International, opinion was divided on the question whether socialists should or should not participate in bourgeois governments. What were the arguments that were adduced for and against? Which groups expressed the one and which the other view? Which view prevailed eventually within the Second International?
  3. What are the rules of rational allocation of productive resources in a socialist society, and how do they differ from the corresponding rules in a capitalist society (a) under conditions of perfect competition and (b) under conditions of monopolistic competition?
  4. State and criticize the Marxian proposition known as the Theory of Increasing Misery (“immiseration”).
  5. Most socialist writers recognize that the transition from the capitalist to the socialist form of life will raise a number of problems that are distinct from the problem of how to run a socialist society when established. What are those “transitional problems” and what methods have been suggested for dealing with them?
  6. What is meant by Reformism? By Revisionism? By Laborism?

 

Source:  Harvard University Archives. Harvard University. Final Examinations, 1853-2001. Box 8, Folder “Final examinations, Winter term, 1943-44”.

Image Source:  Harvard Class Album 1942.

Categories
Exam Questions Harvard

Harvard. Exams for History and Literature of Economics. Schumpeter, 1942-48.

 

Joseph Schumpeter offered his graduate course “History and Literature of Economics since 1776” nine times during the period 1940-1949. The core readings were basically unchanged. In an earlier post I provided the reading list  for 1939-40 along with examinations from the 1939-40 and 1940-41 academic years. The reading list, complete with links to every item, for 1948-49 has also been transcribed and posted.

In this post you will find transcriptions of final examination questions for five of the remaining seven years in the series.

_____________

Final Examination, 1941-42
HARVARD UNIVERSITY
ECONOMICS 113b2

One question may be omitted. Arrange your answers in the order of the questions.

  1. Economists of the most varied types have claimed Adam Smith as patron-saint. How would you describe his general method as revealed in the five books of the Wealth of nations?
  2. If you felt called upon to defend Ricardo’s theory of value, how would you do it?
  3. Marx predicted the breakdown of the capitalist system. What was his main line of argument? Do you think that the course of events in the last decade has verified his theory?
  4. Explain the meaning and use of the theorem usually referred to as Say’s Law.
  5. Senior defined cost of production as the sum of labor and abstinence necessary for production. Do you think this satisfactory?
  6. Does Marshall’s work seem to you to close or to begin a period in the history of economic thought?

 

Source:Harvard University Archives. Final Examinations, 1853-2001 (HUC 7000.28) Box 6, Papers Printed for Final Examinations—History, History of Religions, …, Economics, …, Military Science, Naval Science. June 1942.

_____________

Final Examination, 1942-43
HARVARD UNIVERSITY
ECONOMICS 113b2

Answer any four out of the following five questions.

  1. With the possible exception of Darwin’s Origin of Species, no scientific book has ever equaled the success of the Wealth of Nations. How do you account for this success?
  2. Discuss the Malthusian theory of population and describe its role in the classical system of economic theory.
  3. What do you think of the so-called Ricardian theory of rent?
  4. Criticize the following statement made by Adam Smith: “The price of monopoly is upon every occasion the highest which can be got. The natural price, or the price of free competition, on the contrary, is the lowest which can be taken…for any considerable time.”
  5. The founders of the marginal-utility school evidently believed that they had revolutionized economic theory. What warrant was there for this belief? What do you think their contribution consisted in?

 

Source:Harvard University Archives. Final Examinations, 1853-2001 (HUC 7000.28) Box 7, Papers Printed for Final Examinations—History, History of Religions, …, Economics, …, Military Science, Naval Science. May 1943.

_____________

Final Examination, 1943-44
[not (yet) located]

Course not offered 1944-45

_____________

Final Examination, 1945-46
HARVARD UNIVERSITY
ECONOMICS 113b

Two questions may be omitted. Arrange your answers in the order of the questions.

  1. Explain the meaning and use of “Say’s Law.”
  2. Sketch the history and discuss the merits and demerits of the wage-fund theory.
  3. How did Ricardo explain “profits?”
  4. Choose any economist between 1776 and 1890 with whose work you are familiar, and describe the nature, method, and value of his contribution.
  5. Smith no doubt sponsored what Lord Keynes described as the Fallacy of Cheapness and Plenty. What do you think of the proposition involved?
  6. What relation do you conceive to exist between economists’ theoretical explanations of facts and (a) their political preferences in general and (b) the interests of the social classes to which they belong in particular?

 

Source:Harvard University Archives. Final Examinations, 1853-2001 (HUC 7000.28) Box 11, Papers Printed for Final Examinations—History, Government, Economics, …, Military Science, Naval Science. May 1946.

_____________

Final Examination, 1946-47
HARVARD UNIVERSITY
ECONOMICS 113b

One question may be omitted. Arrange your answers in the order of the questions.

  1. Characterize briefly the nature, and the historical roots, of Adam Smith’s performance.
  2. What was the contribution of Ricardo to economics? And what of it may be said to have survived until today?
  3. State and criticize the various explanations that the English classics offered for what they took to be an indubitable tendency in the rate of profits to fall?
  4. Appraise the role, in the history of economic theory, of Malthus’s Law of Population?
  5. Discuss the validity and the importance of the marginal-utility theory of value.
  6. What do you mean by, and what do you think of, Institutionalism?

 

Source:Harvard University Archives. Final Examinations, 1853-2001 (HUC 7000.28) Box 14, Papers Printed for Final Examinations—History, History of Religions, …, Economics, …, Military Science, Naval Science. May 1947.

_____________

Final Examination, 1947-48
HARVARD UNIVERSITY
ECONOMICS 113b

One question may be omitted. Arrange your answers in the order of the questions.

  1. Discuss the nature and importance of Ricardo’s contributions to economic theory.
  2. Discuss the nature and importance of Senior’s contributions to economic theory.
  3. “Demand for commodities is not demand for labor.” Explain and criticize.
  4. State and analyze the Ricardian theory of Rent.
  5. State and analyze Say’s “Law of Markets.”
  6. Most English classics were votaries of Laissez-faire. What qualifications did they admit, particularly with reference to the labor contract?

 

Source:Harvard University Archives. Final Examinations, 1853-2001 (HUC 7000.28) Box 15, Papers Printed for Final Examinations—History, History of Religions, …, Economics, …, Military Science, Naval Science. May 1948.

_____________

Final Examination, 1948-49
[not (yet) located]

_____________

Image Source: Selection from “Joseph A. Schumpeter and other at dinner table, ca. 1945”, Harvard University Archives HUGBS 276.90p (4).

 

Categories
Economists Harvard Radcliffe Swarthmore

Harvard. Wolfgang Stolper describes his training in letter to Hobart College, 1941

 

This post provides Wolfgang Stolper‘s own description of his academic training, teaching and research interests as of early 1941 in a letter to the President of Hobart College regarding his application for an assistant professorship. Stolper’s Harvard coursework for 1934-37 was transcribed for an earlier post. He was on the job market for the 1941-42 academic year after having taught a wide range of courses at Harvard since completing his Ph.D. in 1938. 

Hobart’s offer ended up being only 73% of the offer he was to receive from  Swarthmore (and which he accepted). From the April 29, 1941 letter from President’s office at Swarthmore:  “I realize that the salary which we are offering you [$3300] is considerably under what you have been receiving this year. It is, however, the equivalent of your combined Harvard and Radcliffe salaries for the past two years…” A follow-up letter from the President of Swarthmore College dated May 7, 1941 confirmed the approval of Stolper’s appointment as Assistant Professor of Economics at Swarthmore for 1941-42 at a salary of $3300 ($900 higher than the Hobart offer).

Also of some interest is the rather casual/modest mention of what ultimately was to become Wolfgang Stolper’s greatest hit: “Right now I am finishing another article on Protection and Real Wages.”

________________

Carbon copy of Wolfgang Stolper’s letter
to President of Hobart College

Wolfgang F. Stolper

19 Ware Street
March 18, 1941

Mr. Brooks Otis
Hobart College
Geneva, N.Y.

 

Dear Mr. Otis,

I am writing this letter to you about my background and training, as you suggested at our meeting on March 17. If you think it desirable either you or I can ask the Harvard appointment office to send you all the documents concerning me.

First about my background. I was born in Vienna in May 1912. There I went to elementary school and through the first three years of high school. We then moved to Berlin where I finished high school (humanistisches Gymnasium) in 1930, and where I also studied law for three semesters. I then went to Bonn, where I studied law and economics in about the same proportions. My economics teachers were Professors Spiethoff, who is now retired, H. v. Beckerath, who is now at Chapel Hill, and Schumpeter who is now in Harvard. From Bonn I went to to Zürich where I wrote a thesis under Professor Eugen Grossmann on the reasons, economic and otherwise, which lead the various nations to defend different economic policies during the World Economic Conference in London in 1933.

In August 1934 I came to Harvard as a Holtzer fellow, and I held a University fellowship during the next year 1935/36. Since 1936 I have been teaching. I took somewhat more than the required eight courses, my main interests within the field of economics being: Theory, Money and Banking, Business Cycles, International Economic Relations, and Building.

In May, 1935 I took my M.A., and in May, 1938 I got my Ph.D.. My thesis was on the British housing boom from 1931-36, and its connection with monetary policy in the widest sense.

My teaching experience has been quite extensive, more so, I believe, than that of most of my colleagues. Besides the usual complement of tutees and the principles course I have been giving half of the lectures in the course on International Economic Relations [Econ 43a,  Report of the President of Harvard College 1938-39, p. 98;  Report of the President of Harvard College 1939-40, p. 99;  Report of the President of Harvard College 1940-41, p. 58,], and I am this year assisting Professors Haberler and Hansen in the course on Business Cycles [Econ 45a, Report of the President of Harvard College 1940-41, p. 58]. In Radcliffe College I am also giving a section of the course on principles, half of the course on International Economic Relations, and also half of the course on Business Cycles [Radcliffe College. Courses of Instruction 1940-41, pp. 43, 45.]. This year I was also asked to give a University Extension course on International Economic Relations [Report of the President of Harvard College 1940-41, p. 347].

The list of my publications does not look too impressive. I have published a number of short book reviews in the American Economic Review, a short theoretical article in the Quarterly Journal. My thesis which I have revised and brought up to date will be published by the Harvard University Press. I also just signed a contract with Blakiston Co. to write a text on International Trade together with my friend, Dr. H.K. Heuser who is Professor at the Fletcher School of Law and Diplomacy. Right now I am finishing another article on Protection and Real Wages.

Perhaps I should add that I am married and have a ten-year old son.

I hope that this information is what you wanted. My teachers and senior colleagues will of course be glad to give you any information about me which you might want. I need hardly add that I am very interested in the position, and that I, therefore, hope very much to hear from you again in the not too distant future.

Yours very sincerely,

________________

Job Offer to Wolfgang Stolper from Hobart College

HOBART COLLEGE
Geneva, New York

April 29, 1941

The President

 

Mr. Wolfgang F. Stolper
19 Ware Street
Cambridge, Massachusetts

 

Dear Mr. Stolper:

I regret to think that more than a month has passed since I talked with you at Cambridge. Doubtless you have gathered that we had entirely lost interest in you in the interim. That is, however, by no means the case. Since we have to appoint three men at this time, a sociologist, a political scientist and an economist we have, necessarily, had to proceed rather slowly. Our major preoccupation so far has not been Economics simply because we have had to concentrate on one thing at a time. We have now, however, reached a point where the Economics appointment is directly concerning us.

I am writing this letter to you to find out more definitely what your expectations would be. The plain fact is that we find that we cannot offer the salary that we should like to. The best thing that we can do for next year in Economics is an Assistant Professorship at twenty-four hundred dollars ($2400.00). I might also add, as I think I said to you personally, that anyone appointed to this position would have considerable freedom in the choice of courses and in the teaching, and a considerable opportunity to influence the operation and planning of the whole Social Science curriculum here in cooperation with his colleagues in Sociology and Political Science,–that is, of course, if he cared to do so.

If it is not too much trouble, would you drop me a line stating whether you would still be interested in the job as outlined above?

Very truly yours,
[signed]
Brooks Otis

BO/bg

________________

Carbon copy of Stolper letter
declining Hobart offer

Wolfgang F. Stolper

19 Ware Street
May 2, 1941

Mr. Brooks Otis
Hobart College
Geneva, N.Y.

 

Dear Mr. Otis,

Thank you very much for your letter of April 29. I regret very much that I have to reject your offer to come to Hobart next year since I have just accepted a position as Assistant Professor at Swarthmore College.

I was very glad to have had a chance of meeting you, and I hope very much that this letter will not be the end of our relationship.

Very truly yours,

 

Source:Duke University. David M. Rubenstein Rare Book & Manuscript Library. Economists’ Papers Archive. Wolfgang F. Stolper Papers, Box 23, Folder “[illegibly marked]”

Image Source: Wolfgang F. Stolper from  John Simon Guggenheim Memorial Foundation (Fellow, 1947).

Categories
Exam Questions Harvard

Harvard. Year-end exams. Money, Banking, Commercial Crises. Young, 1921-27

 

Today’s artifacts come from the roaring ’20s. Besides his courses in economic theory, Allyn A. Young taught a year long course at Harvard, “Money, Banking and Commercial Crises”. Before presenting enrollment figures and the exams for Young’s Economics 3, I have assembled a chronology that identifies the course instructors over the entire period 1911-1946. Links are provided to the related artifacts that have been transcribed here at Economics in the Rear-view Mirror. 

The chronology is followed by Young’s course description for 1924-25. Presumably there was a mid-year exam for the course, but these were not included in the printed collection of final course examinations. It is possible that the questions have been limited to the second-semester’s course content. This is something that definitely deserves checking.

___________________

Chronology of the Harvard economics course
“Money, Banking and Commercial Crises”

This two semester course was the product of merging the one semester course “Commercial Crises and Cycles of Trade” (Economics 12) with the two semester sequence “Money” and “Banking and Foreign Exchange” (Economics 8a and 8b, respectively).

The new course “Money, Banking, and Commercial Crises” (Economics 8, then 3, and later 41) was a staple of economics course offerings for the next 35 years.

Economics 8

1911-12 taught by E.E. Day

Economics 3

1912-13, 1913-14 taught by E.E. Day.

Money, Banking, and Commercial Crises (1914-15) taught by Benjamin M. Anderson.

1915-16 taught by Norman John Silberling

Money, Banking, and Commercial Crises (1917-18) taught by Benjamin M. Anderson.

1918-19, 1919-20 taught by A. E. Monroe.

1920-21 through 1926-27 taught by Allyn A. Young. Year-end exams transcribed below.

1927-28 through 1931-32 taught by John H. Williams

1932-33 taught by John H. Williams, Joseph Schumpeter and Lauchlin Currie.

1933-34 [course title: Money, Banking, and Cycles] Seymour Harris

1934-35, 1935-36 taught by John H. Williams and Seymour Harris

Economics 41

1936-37  taught by John H. Williams and Seymour Harris

Money, Banking, and Commercial Crises (1937-38) John H. Williams and Richard V. Gilbert.

1938-39 to 1941-42 taught by John H. Williams and Seymour Harris

1942-43, 1943-44 taught by Alvin Hansen and John H. Williams

1944-45 first semester taught by Schumpeter, second semester by Hansen and Williams

1945-46 Economics 41 morphed back into a two semester course “Money and Banking” taught by John H. Williams with a new one term course “Business Cycles” taught by Alvin Hansen.

________________________

Course Description, 1924-25

[Economics] 3. Money, Banking, and Commercial Crises. Mon., Wed., Fri., at 2. Professor Young.

In this course money and credit will be studied with special reference to the part they play in the present economic system. The principal problems of public policy with respect to the control of money and banking will be discussed. Foreign exchange, organized speculation in its relation to the money market, and the characteristic phenomena of commercial crises will be considered in some detail. The course will be conducted by means of lectures, discussions, frequent short reports or exercises on assigned topics, and (in the second half-year) a thesis based on work in the library. Certain subjects, such as the monetary and banking history of the United States, will be covered almost wholly by assigned reading, tested by written papers.

Source:  Division of History, Government and Economics 1924-25 published in Official Register of Harvard University, Vol. 21, No. 22 (April 30, 1924), p. 67.

_______________________

Enrollment, 1920-21

[Economics] 3. Professor Young —Money, Banking, and Commercial Crises.

Total 148: 6 Graduates, 34 Seniors, 67 Juniors, 26 Sophomores, 3 Freshmen, 30 Others.

Source:  Harvard University. Report of the President of Harvard College 1920-21, p. 19.

 

Year-end examination, 1920-21
HARVARD UNIVERSITY
ECONOMICS 3

  1. What is a dollar?
  2. In what manner and why were bank reserves inelastic under the national banking system? What were the consequences?
  3. Discuss the relation of overproduction to crises, distinguishing carefully different types of overproduction.
  4. Outline the sequence of events in a typical business cycle.
  5. Define: federal reserve bank note, gold-exchange standard, “value of money.”
  6. In what different ways may federal reserve notes be issued?
  7. Explain and discuss the “equation of exchange.”
  8. Describe and explain the dominating position the London money market held before the war.

 

Source:  Harvard University Archives. Examination Papers 1921 (HUC 7000.28, No. 63), Papers Set for Final Examinations [in] History, Church History,…,Economics,…, Fine Arts, Music. June, 1921, p. 56.

________________________

Course announcement, 1921-22

[Economics] 3. Money, Banking, and Commercial Crises

Mon., Wed., Fri., at 1.30. Professor Young.

Source:  Harvard University, Announcement of the Courses of Instruction Offered by the Faculty of Arts and Sciences for the Academic Year, 1921-22 (Third Edition),p. 109.


Year-end examination, 1921-22
HARVARD UNIVERSITY
ECONOMICS 3

  1. Draw up a statement showing the condition of a national bank. Explain the meaning of the various items.
  2. Under what conditions is a large surplus an indication of a bank’s strength? How may it be an indication of weakness?
  3. To what classes of persons are rising prices advantageous? To what classes are they disadvantageous?
  4. Define: gold exchange standard, banker’s acceptance, finance bill, bimetallism, index number.
  5. What do you take to have been the causes of the fall of prices between 1874 and 1896?
  6. Why were “surplus reserves” under the national banking system normally exceedingly small?
  7. State and explain the Ricardian theory of gold movements. Are the recent movements of gold from Europe to the United States explainable by the Ricardian principle?
  8. What relation was there between the Bank Act of 1844 and the controversies of the restriction period?
  9. If the weight of the gold dollar were reduced by half would prices be doubled? Explain your reasoning.
  10. “The bulk of the acceptance business arising out of the foreign trade of the entire world has for many years been conducted in London.” Explain what this statement means and why it is true.

Final. 1922

 

Source:  Harvard University Archives. Examination Papers 1922 (HUC 7000.28, No. 64), Papers Set for Final Examinations[in] History, Church History,…,Economics,…, Social Ethics, Education. June, 1922.

________________________

Enrollment, 1922-23

[Economics] 3. Professor Young—Money, Banking, and Commercial Crises.

Total 129: 6 Graduates, 33 Seniors, 75 Juniors, 11 Sophomores, 1 Freshman, 3 Others.

Source:  Harvard University. Report of the President of Harvard College 1922-23, p. 92.


Year-end examination, 1922-23
HARVARD UNIVERSITY
ECONOMICS 3

  1. Define: money of account, standard of deferred payments, inflation, gold-exchange standard, discounting.
  2. Give an account of the life-history of a typical commercial long bill of exchange, as used in international trade.
  3. Discuss the nature and significance of the par of exchange between two countries when one has a gold standard and the other has (a) a gold standard, (b) a silver standard, (c) inconvertible paper.
  4. Is New York City likely to become the center of the world’s foreign exchange markets? Discuss.
  5. In what ways are federal reserve notes and clearing-house loan certificates alike? In what ways are they unlike?
  6. Professor W. C. Mitchell holds that prosperity breeds a crisis because of (a) the gradual increase in the costs of doing business, and (b) the accumulating tension of the investment and money markets. Explain and discuss.
  7. Was the federal reserve system responsible for the rise of prices between 1917 and 1920 and for the subsequent drop? Discuss.
  8. In what ways do the federal reserve banks effect (a) regional and (b) national clearings?
  9. On what grounds is it generally held that a larger use of bank acceptances in this country is desirable?

Final. 1923.

 

Source:  Harvard University Archives. Examination Papers 1923 (HUC 7000.28, No. 65), Papers Printed for Final Examinations [in] History, History of Religions,…,Economics,…, Social Ethics, Anthropology. June, 1923.

________________________

Enrollment, 1923-24

[Economics] 3. Professor Young—Money, Banking, and Commercial Crises.

Total 119: 2 Graduates, 25 Seniors, 81 Juniors, 5 Sophomores, 1 Freshman, 5 Others.

Source:  Harvard University. Report of the President of Harvard College 1923-24, p. 106.

 

Year-end examination, 1923-24
HARVARD UNIVERSITY
ECONOMICS 3

Answer nine questions.

  1. Explain the first and either the second or the third of these theories of the business cycle: (1) the “banking theory”; (2) Hobson’s theory of over-saving; (3) Fisher’s theory of the lagging adjustment of interest.
  2. “It thus appears that the Bank of England’s official rate is often through long periods a mere empty symbol, leaving no actual relation to the real price of money in London; and only becomes effective, and a factor in the monetary position when…” When?
  3. Draw up a statement showing the principal items which enter into the balance of payments.
  4. What conditions must be fulfilled if New York is to become the center of the world’s foreign exchange markets?
  5. State and discuss the doctrine of purchasing-power parity.
  6. Discuss the open-market operations of the federal reserve banks, with special reference to (a) the provisions of the law, (b) the purposes of such operations, (c) their relation to possible changes in prevalent types of commercial paper.
  7. Why did national bank notes constitute an inelastic currency? in just what manner do federal reserve notes constitute an elastic currency?
  8. Discuss the effect of organized speculation on prices, taking account of the fact that different types of price variations cover different periods of time.
  9. G. Moulton lists as “fallacies,” (1) the notion that a nation’s capacity to pay a foreign debt (such as reparations) is measured by the excess of its annual production over its annual consumption, and (2) the notion that a country can pay such a debt by selling securities to other countries. Do you agree? Explain.
  10. “In the main, banks do not lend their deposits, but rather, by their own extensions of credit, create the deposits.” Explain.

Final. 1924.

 

Source:  Harvard University Archives. Examination Papers 1924 (HUC 7000.28, No. 66), Papers Printed for Final Examinations [in] History, History of Religions,…, Economics,…, Psychology, Social Ethics. June, 1924.

________________________

Enrollment, 1924-25

[Economics] 3. Professor Young—Money, Banking, and Commercial Crises.

Total 111: 1 Graduate, 22 Seniors, 72 Juniors, 12 Sophomores, 1 Freshman, 3 Others.

Source:  Harvard University. Report of the President of Harvard College 1924-25, p. 75.

 

Year-end examination, 1924-25
HARVARD UNIVERSITY
ECONOMICS 3

Answer eight questions.

  1. Some writers hold that business cycles are caused by the expansion and contraction of bank credit. Why and how, in their view, does bank credit expand and contract?
  2. “A country can pay a foreign debt only by exporting more than it imports.” Explain and discuss critically.
  3. What was the major defect of the old national banking system?
  4. Define: rediscount, trust company, par collections, gold standard, purchasing power parity.
  5. “The Bank of England has power to exert a decisive influence over the magnitude of the gold movements to and from England.”—Furniss.
  6. What are the distinguishing characteristics (economic or legal, not physical characteristics) of the following types of money: silver dollars, United States notes, national bank notes, federal reserve notes?
  7. What are the prerequisites to the stabilizing of a depreciated paper currency?
  8. In what measure was the federal reserve system responsible for the rapid rise of prices in 1919 and 1920 and for the subsequent collapse?
  9. The federal reserve banks hold nearly $3,000,000,000 in gold, amounting to about 75 per cent of their liability on account of deposits and note issues combined, and constituting a large idle investment. Under what conditions would a considerable part of this gold be exported to other countries?

Final. 1925.

 

Source:  Harvard University Archives. Examination Papers 1925 (HUC 7000.28, No. 67), Papers Printed for Final Examinations [in] History of Science, History, …, Economics,…, Anthropology, Military Science. June, 1925.

________________________

Enrollment, 1925-26

[Economics] 3. Professor Young—Money, Banking, and Commercial Crises.

Total 110: 31 Seniors, 64 Juniors, 8 Sophomores, 1 Freshman, 6 Others.

Source:  Harvard University. Report of the President of Harvard College 1925-26, p. 77.

 

Year-end examination, 1925-26
HARVARD UNIVERSITY
ECONOMICS 3

Answer eight questions.

  1. Define deposits, discount, monetary standard, bimetallism.
  2. Formulate the “quantity theory” in any way that you prefer, and discuss it critically.
  3. A Brazilian firm draws a 90-day bill upon a London banker on account of a shipment of coffee to Boston.

(1) Why should the London bill be preferred to a bill upon New York or Boston?
(2) What is done with the bill after it reaches London?
(3) How is the bill finally settled?

  1. Some writers hold that when a government issues inconvertible paper money it obtains what is virtually a “forced loan.” Others hold that such an issue is more like taxation. What is your opinion, and why?
  2. Give an account of one of the following:

The socialist theory of crises.
Hobson’s theory of over-saving.
The “banking theory” of crises.

  1. Explain briefly the meaning of any two of the following phrases:

Par-collections controversy.
Open market policy.
Gold settlement fund.
Rediscounting

  1. Compare the Bank of England and either the Bank of France or the Reichsbank with respect to

(a) restrictions on note issue;
(b) discount policy.

  1. Was the federal reserve system responsible for the inflation of 1919-20 and the ensuing collapse? Explain.
  2. Just why, in your opinion, did the mark (or the franc, or the greenback) depreciate?

Final. 1926.

 

Source:  Harvard University Archives. Examination Papers 1926 (HUC 7000.28, No. 68), Papers Printed for Final Examinations [in] History, History of Religions, …, Economics,…, Social Ethics, Military Science. June, 1926.

________________________

Enrollment, 1926-27

[Economics] 3. Professor Young and Mr. Marget.—Money, Banking, and Commercial Crises.

Total 125: 2 Graduates, 27 Seniors, 74 Juniors, 14 Sophomores, 2 Freshmen, 6 Others.

Source:  Harvard University. Report of the President of Harvard College 1926-27, p. 74.

 

Year-end examination, 1926-27
HARVARD UNIVERSITY
ECONOMICS 3

Answer eight questions.

  1. Explain and discuss critically some form of the “banking” or “credit” theory of business cycles.
  2. “If prices are rising” Hawtrey observes, “the mere holding of commodities in stock yields an additional profit over and above the usual dealer’s percentage on the turn-over. If traders are to be deterred from borrowing money to buy commodities, the rate of discount must be high enough to offset the additional profit. But, it may be asked, how is this possible when prices are rising at the rate of 30 per cent per annum?” Hawtrey’s answer? Your own?
  3. Discuss critically either (a) Fisher’s proposals for stabilizing the price level, or (b) proposals for attaining the same end by controlling the supply of bank credit.
  4. Select two of the following and discuss their significance as “causes” of the depreciation of inconvertible paper money: (1) excessive quantity; (2) ultimate redemption uncertain; (3) unbalanced budget; (4) adverse balance of foreign payments; (5) speculation.
  5. Define: rediscounts, purchasing-power parity, invisible exports, monetary standard, par collections.
  6. Compare the note-issue system of the Bank of England (as established by the Act of 1844) with the note-issue system of the federal reserve banks, with particular reference to (a) separation of “banking” and “issue” departments, and (b) the type of assets by which the notes are “covered.”
  7. In what way or ways do purchases and sales of government securities in the New York money market by the federal reserve banks affect the state of that market?
  8. If you were Dictator of France, and took account of considerations of justice as well as of expediency, would you plan to stabilize the franc at its present (gold) value? Or would you plan for a gradual recovery of its pre-war value? Why?
  9. Discuss the relation of international gold movements to changes of (a) relative price levels, (b) relative discount rates.

Final. 1927.

 

Source:  Harvard University Archives. Examination Papers 1927 (HUC 7000.28, No. 69), Papers Printed for Final Examinations [in] History, History of Religions, …, Economics,…, Social Ethics, Military Science. June, 1927.

Image Source: Allyn Young in Harvard Classbook 1925.

 

 

Categories
Cambridge Chicago Columbia Economic History Economists Germany Harvard NBER Stanford

Chicago. Friedman memo regarding Karl Bode and Moses Abramovitz, 1947

 

In the following 1947 memo from Milton Friedman to T.W. Schultz we can read two talent-scouting reports on potential appointments for the University of Chicago economics department. One candidate, Karl Bode had been vouched for by Allen Wallis, a trusted friend and colleague of Milton Friedman, but we can easily read Friedman’s own less than enthusiastic report on the meager published work examined, certainly compared to Friedman’s glowing report for his friend from Columbia student days, Moses Abramovitz. But comparing the publications listed in the memo, I certainly wouldn’t fault Friedman’s revealed preference for Abramovitz.

Abramovitz went on to have a long and distinguished career at Stanford and Bode left Stanford for government service with his last occupation according to his death certificate “Planning Director, Agency for International Development (A.I.D.)”

Since Karl Bode turned out to have cast a relatively short academic shadow, I have appended some biographical information about him at the end of this post. But for now just the vital dates: Karl Ernst Franz Bode was born November 24, 1912 in Boennien, Germany and he died March 18, 1981 in Arlington, VA.

__________________

Milton Friedman on Bode and Abramovitz

January 10, 1947

[To:] Mr. Schultz, Economics
[From:] Mr. Friedman, Economics
[Re:] Staff appointments

In connection with staff appointments, I thought it might be helpful if I put down on paper for you the information I have on two persons whose names I have casually mentioned: Karl Bode and Moses Abramovitz.

  1. Karl Bode (Assoc. Prof. of Economics, Stanford)

I know about Bode primarily from Allen Wallis. Allen considers him absolutely first-rate in all respects and recommends him very highly.

Bode, who is now in his early thirties, was born in Germany and, though Catholic of Aryan descent, and the holder of a highly-prized governmental fellowship, left Germany almost immediately after Hitler’s accession. He went first to Austria, then to Switzerland, where he took his Ph.D., in 1935, then to England, where he studied at Cambridge and at the London School. Bernard Haley met him while at Cambridge, was highly impressed with him, and induced him to come to Stanford, where he has been since 1937. He has been on leave of absence since early 1945, first with the Tactical Bombing Survey, then with the Allied Military Government in Berlin. He is expected back sometime this summer.

At Stanford, Bode is responsible for American and European Economic History, and, in addition, has taught advanced courses in Economic Theory. His original interest was in International Trade. He has a contract to write a text on Economic History, but I do not know whether on American or European Economic History.

I have obtained a list of his publications, most of which are fragments or reviews. Three of more general interest are:

(a) A. W. Stonier: “A New Approach to the Methodology of the Social Sciences”, Economica, Vol. 4, p. 406-424, Nov., 1937.

(b) “Plan Analysis and process analysis: AER, 33-348-54, June 1943.

(c) “A Note on the Mathematical Coincidence of the instantaneous and the serial multiplier”, Review of Economic Statistics, 26: 221-222, Nov. 1944.

I have read these. They are too slight to permit a reliable and comprehensive judgment about his capacities; but they are sufficient to demonstrate a clear, logical mind.

Allen tells me that Schumpeter, Haberler, Howard Ellis, and of course, the Stanford people all know him and could provide evidence about his abilities.

 

  1. Moses Abramovitz (member of research staff in charge of business cycle unit, National Bureau of Economic Research.)

Abramovitz got his bachelor’s at Harvard, his Ph.D. at Columbia. He has done some part-time teaching of Theory at Columbia. During the war he was with the Office of Strategic Services, where he worked on foreign economic conditions. He was a member of the reparations commission staff at both the Moscow and Paris Conferences.

Abramovitz and I were fellow graduate students at Columbia, and I have known him rather well ever since. I think him extremely capable, with an excellent mind, broad interests, and an extraordinary capacity for forming a sound judgment from conflicting evidence.

His academic and private research background is mostly in Economic Theory and Business Cycles; but the war years gave him a considerable background, and generated a real interest, in foreign economic relations.

Some of his writings are:

Selected Publications:

An Approach to a Price Theory for a Changing Economy, Columbia University Press, 1939.

Monopolistic Selling in a Changing Economy, Q.J.E., Feb., 1938.

Saving vs Investment: Profits vs Prosperity?Supplement on papers relating to the TNEC, Am. Econ. Rev., June, 1942.

Book on Cyclical behavior of inventories completed and scheduled to be published shortly by Nat’l Bureau of Economic Research.

M.F.

ab

* * * * *

PUBLICATIONS OF KARL BODE

A new approach to the methodology of the social sciences. (With A.W. Stonier): Economica, vol. 4, pp. 406-424, November, 1937.

Prosperität und Depression: Zeitschrift für Nationalökonomie, vol. 8, pp. 597-614, December, 1937.

Review of: Plotnik, M.J. Werner Sombart and his type of economics. 1937. American Economic Review, 28: 522-523, September, 1938.

Review of: Sombart, Werner. Weltanschauung, Wissenschaft und Wirtschaft. 1938. Ibid., 28: 766, December, 1938.

The acceptance of defeat in Germany: Journal of abnormal and social psychology, 38: 193-198, April, 1943.

Plan analysis and process analysis: American Economic Review, 33: 348-354, June, 1943.

Review of: Day, C. Economic Development in Europe. 1942:Journal of economic History, 2: 225-227, November, 1942.

Catholics in the postwar world: America, 71: 347-348, July, 1944

Economic aspects of morale in Nazi Germany: Pacific Coast Economic Association: Papers, 1942. pp. 29-34, 1943.

Reflections on a reasonable peace: Thought, 19: 41-48, March, 1944

Review of: Dempsey, B.W. Interest and usury. 1943: Ibid., 18: 756-758, December, 1943.

German reparations and a democratic peace: Thought, 19: 594-606, December, 1944

A note on the mathematical coincidence of the instantaneous and the serial multiplier: Review of Economic Statistics, 26: 221-222, November, 1944.

 

Source:Hoover Institution Archives. Papers of Milton Friedman, Box 79, Folder 1 “University of Chicago, Minutes. Economics Department 1946-1949”.

__________________

Karl F. Bode
AEA 1969 Directory of Members, p. 41.

Bode, Karl F., government; b. Germany, 1912; student, U. Bonn-Germany, 1931-33, U. Vienna-Austria, 1933-34; Ph.D., U. Bern-Switzerland, 1935; Cambridge-England, 1935-37. DOC.DIS. The Concept of Neutral Money, 1935. FIELDS 2abc, 1c, 4a. Chief, Regional Organization & Program Staff, Intl. Cooperation Adm., 1955-60, asst. dep. dir. for planning, 1960-62; chief, Planning Assistance & Research Div., Agy. for Intl. Dev., 1962-67; dir., Research, Evaluation & Information Retrieval, Agy. for Internat. Dev. since 1967. ADDRESS Vietnam Bur., Agy. for Internat. Dev., Dept. State, Washington, DC 20523.

__________________

 Haberler Report of Mises’s Private Seminar

Regular participants of the seminar were several members of the Mont Pelerin Society – notably Hayek, Machlup, the late Alfred Schutz and in the very early days, John V. Van Sickle. Visiting scholars regarded it a great honor to be invited to the seminar – among them Howard S. Ellis (University of California), Ragnar Nurkse (late Professor of Economics in Columbia University, New York) whose untimely death occurred three years ago, Karl Bode (later in Stanford University and now in Washington), Alfred Stonier (now University College in London), and many others. There was Oskar Morgenstern (now Princeton University), the late Karl Schlesinger and Richard Strigl, two of the most brilliant economists of their time…the unforgettable Felix Kaufmann, philosopher of the Social Sciences in the broadest sense including the law and economics – he also wrote a much debated book on the logical foundation of mathematics – who after his emigration in 1938 joined the Faculty of the New School for Social Research in New York where he taught with great success until his premature death twelve years ago.

Source: Mises’s Private Seminar: Reminiscences by Gottfried Haberler. Reprint from The Mont Pelerin Quarterly, Volume III, October 1961, No. 3, page 20f. Posted at the Mises Institute website.

__________________

 From the Preface of Felix Kaufman’s 1936 book

For the critical editing of the manuscript and of the galleys, I wish to thank most heartily a number of friends in various countries, expecially Dr. Karl Bode, presently of St. John’s College, Cambridge and Dr. Alfred Schütz of Vienna. Dr. Bode has also taken upon himself the great labor of preparing both indexes.

Source: Felix Kaufmann. Theory and Method in the Social Sciences. [English translation of Methodenlehre der Sozialwissenschaften. Wien: Julius Springer, 1936.] from Felix Kaufmann’s Theory and Method in the Social Sciences, Robert S. Cohen and Ingeborg K. Helling (eds.). Boston Studies in the Philosophy and  History of Science, 303. Springer: 2014.

__________________

 Reports from The Stanford Daily

The Stanford Daily, Volume 93, Issue 47, 29 April 1938

Several distinguished scholars from other universities will join the Stanford faculty next year…Dr. Karl Franz Bode, formerly on the faculty of St. John’s College, Cambridge University, England, was appointed assistant professor of economics to succeed Dr. Donald M. Erb who was appointed president of the University of Oregon….

 

The Stanford Daily, Volume 100, Issue 02, 23 September 1941, p. 1.

Econ Department Changes Classes… History of Currency Problems, 118, will he given in fall quarter rather than in the spring quarter. It is a five-unit course, taught MTWThF at 11 a.m. in Room 200Q by Karl F. Bode. Economics 1 and 2 are prerequisites….

 

The Stanford Daily, Volume 103, Issue 86, 28 May 1943, p. 1.

Wilbur Names New Faculty Promotions. Promotions and appointments of faculty members for the academic year 1943-1944 were announced yesterday by Chancellor Ray Lyman Wilbur. … Those promoted from assistant professor to associate professor are … Dr. Karl F. Bode, economics….

 

The Stanford Daily, Volume 111, Issue 20, 7 March 1947, p. 3

President Donald B. Tresidder yesterday announced 37 faculty promotions. The promotions include 11 faculty members to full professorships, six to associate professorships, and two to assistant professorships, together with promotion of 18 members of the clinical faculty at the Stanford School of Medicine in San Francisco….

To professorships … Karl F. Bode, in economics…

 

The Stanford Daily, Vol 119, Issue 7, 13 February 1951, p. 1.

Dr. Karl F. Bode, Stanford economics professor on leave for government duty in Germany, has been appointed deputy economic adviser, Office of Economic Affairs, it has been announced by the office of the U.S. High Commissioner for Germany. Dr. Bode will be stationed in Bonn, Germany. He has been acting chief of the program division in the Office of Economic Affairs.

 

Image Source: Karl Bode from the 1939 Standford Quad.

Categories
Austria Economists

Austrian economist mugshots. Österreichische Nationalbibliothek

 

The Austrian National Library (Österreichische National Bibliothek) has impressive digital resources online that include books, E-books, photographs, newspapers, and journals etc. Below I provide slightly edited copies and links to photographs of nine Austrian economists of note. I have not conducted an exhaustive search, but thought visitors to Economics in the Rear-view Mirror might find the sample below useful for presentation purposes. 

Rudolf Auspitz (1837-1906)
Carl Menger (1840-1921)
Eugen von Böhm-Bawerk (1851-1914)
Friedrich von Wieser (1851-1926)
Ludwig von Mises (1881-1973)
Josef Alois Schumpeter (1883-1950)
Friedrich von Hayek (1899-1992)
Gottfried von Haberler (1900-1995)
Oskar Morgenstern (1902-1977)

 

 ________________________

Rudolf Auspitz (1837-1906)

Link to Österreichische Nationalbibliothek record.

Link to Österreichische Nationalbibliothek record.

 

 

________________________

Carl Menger (1840-1921)

Link to Österreichische Nationalbibliothek record.

 

Link to Österreichische Nationalbibliothek record.

 

 

________________________

Eugen von Böhm-Bawerk (1851-1914)

[1896]

Link to Österreichische Nationalbibliothek record.

 

Link to Österreichische Nationalbibliothek record.

Link to Österreichische Nationalbibliothek record.

 

 

Link to Österreichische Nationalbibliothek record.

 

 

________________________

Friedrich von Wieser (1851-1926)

28 February 1914

Link to Österreichische Nationalbibliothek record.

 

Link to Österreichische Nationalbibliothek record.

 

Link to Österreichische Nationalbibliothek record.

 

 

________________________

Ludwig von Mises (1881-1973)

1935

Link to Österreichische Nationalbibliothek record.

 

 

________________________

Josef Alois Schumpeter (1883-1950)

State Secretary for Finance in the Second Cabinet of Renner.

Link to Österreichische Nationalbibliothek record.

Link to Österreichische Nationalbibliothek record.

 

 

Welt-Press-Photo, 1920.

Link to Österreichische Nationalbibliothek record.

 

 

________________________

Friedrich von Hayek (1899-1992)

Ca. 1930.

Link to Österreichische Nationalbibliothek record.

 

 

________________________

Gottfried von Haberler (1900-1995)

Link to Österreichische Nationalbibliothek record.

 

________________________

Oskar Morgenstern (1902-1977)

Link to Österreichische Nationalbibliothek record.

Categories
Exam Questions Harvard

Harvard. Graduate economic theory exams. Taussig, 1930-35

 

Today I am relieved to post the final batch (1930-1935) of enrollment data and examination questions for Frank W. Taussig’s core economic theory course. All in all nearly a half-century run for Harvard’s Grand Old Man.

Previous batches of transcribed exams are provided via the links below.

Examinations for 1887-90
Examinations for 1891-94
Examinations for 1897-1900
Examinations for 1904-09
Examinations for 1911-14
Examinations for 1915-17
Examinations for 1918-19 [Bullock and Carver]
Examinations for 1920-22
Examinations for 1923-25
Examinations for 1926-30

____________________________________

1930-31

Course Enrollment: Economics 11
1930-31

[Economics] 11. Professor Taussig.—Economic Theory

Total 58: 50 Graduates, 1 Senior, 7 Radcliffe.

 

Source: Harvard University. Reports of the President and Treasurer of Harvard College, 1930-31, p. 77.

 

1930-31
HARVARD UNIVERSITY
ECONOMICS 11
Mid-year Examination

Arrange your answers in the order of the questions.
One question may be omitted.

  1. In an examination paper set at Harvard College in 1876 the following question appears: “What is the error in the proposition that high wages make high prices?”
    What answer would have been expected from a student at that time? What answer would you give now?
  2. “The latent influence by which the values of things are made to conform in the long run to the cost of production is the variation that would otherwise take place in the supply of the commodity. The supply would be increased if the thing continued to sell above the ratio of its cost of production, and would be diminished if it fell below that ratio. But we must not therefore suppose it to be necessary that the supply should actually be either diminished or increased. . . . There is no need that there should be any actual alteration of supply; and when there is, the alteration, if permanent, is not the cause, but the consequence of the alteration in value. If, indeed, the supply could not be increased, no diminution in the cost of production would lower the value: but there is by no means any necessity that it should. The mere possibility often suffices.”
    Is this in accord with Mill’s analysis of demand and supply? with Marshall’s? with business experience?
  3. Can you distinguish between “supply price” and “expenses of production” in the following cases:
    1. the temporary equilibrium of supply and demand;
    2. accountants’ figures of cost for agricultural produce;
    3. accountants’ treatment of depreciation in the accounts of a manufacturing enterprise.
  4. In an examination paper set at Cambridge University, England, in 1929, the following appears: “From the point of view of economic principle, analyze the return obtained to-day from fen land drained in the seventeenth century?”
    What answer would Ricardo or Mill have given? What answer would be expected now from a student in Cambridge, England? What from a student in Cambridge, Mass.?
  5. (1) Marshall’s final conclusion as to the tenability of a distinction between interest and rent.
    (2) The following passages:

“The deepest and most important line of cleavage in economic theory” [is] “the distinction between the quasi-rents which do not, and the profits which do, directly enter into the normal supply prices of produce for periods of moderate length.”
“When the artisan or professional man has once obtained the skill required for his work, a part of his earnings are for the future really a quasi-rent of the capital and labour invested in fitting him for his work, in obtaining his start in life, his business connections, and generally his opportunity for turning his faculties to good account; and only the remainder of his income is true earnings of effort. But this remainder is generally a large part of the whole. And here lies the contrast. For when a similar analysis is made of the business man, the proportions are found to be different: in his case the greater part is quasi-rent.”

Is there inconsistency, apparent or real?

  1.    a.  Adam Smith’s remark, that the division of labor is limited by the extent of the market, has been said to state the gist of all there is to be said about external economies.
    1. It has been said, again, that the only internal economies which signify as regards economic theory are those accruing from the growth of production on a large scale.
    2. “If a commodity obeys the law of increasing return, an increase of demand causes much more of it to be produced, — more than if the commodity obeyed the law of constant return, — and at the same time lowers its price. . . . This line of reasoning has been thought by some writers to lend support to the claim that a Protective duty on manufactured imports in general increases the home market for those manufactured goods; and, by calling into play the Law of Increasing Return, ultimately lowers their price to the home consumer.”
    3. Consider these, separately or as a whole.
  1.     a. “Let us suppose that every one owns whatever capital he uses . . . and is not only of equal capacity, but of equal willingness to work, and does in fact work equally hard; also that all work is unskilled, — or rather, unspecialized in this sense, that if any two people were to change occupations, each would do as much and as good work as the other one had done.”
    1. “Let us suppose that labor is not of one industrial grade, but of several; that parents always bring up their children to an occupation of their own grade; that they have a free choice within that grade, but not outside it. Let us suppose, further, that the increase of population in each grade is governed by other than economic causes; it may be fixed, or may be influenced by changes in custom, in moral opinion, etc.”
    2. What would govern relative wages under each of these suppositions? What would govern the value of goods? Which supposition underlies Marshall’s conclusions on the relation between wages and value?

 

 

1930-31
HARVARD UNIVERSITY
ECONOMICS 11
Final Examination

Answers questions 1, 2, 3 briefly; 4 and 5 more at length.

  1. Jevons remarked: “Capital, as I regard it, consists merely in the aggregate of those commodities which are required for sustaining laborers of any kind or class engaged in work. . . . The single and all-important function of capital is to enable the laborer to await the result of any long-lasting work, — to put an interval between the beginning and the end of an enterprise.”
    Wherein does this resemble, wherein differ from, the view of Ricardo? Böhm-Bawerk? Marshall? Clark?
  2. Public encouragement or discouragement for industries of increasing, constant, or decreasing returns, — wherein the analysis of Pigou resembles that of Marshall, wherein differs.
  3. The bearing on the national dividend and its maximization, of the price structure obtaining under —

Simple competition,
Simple monopoly,
Joint supply,
Discriminating monopoly.

  1. Are there grounds for considering “profits” as an element in distribution different from wages, interest, rent?
  2. The doctrine that wages are determined by the marginal productivity of labor; the grounds on which it rests; and the aid it may give on such questions as the (1) basis of fair wages in the arbitration of industrial disputes, and the (2) effect on contractual wages of a compulsory system of social insurance (accident, sickness, old age, unemployment).

____________________________________

1931-32

Course Enrollment: Economics 11
1931-32

[Economics] 11. Professor Taussig.—Economic Theory

Total 48: 38 Graduates, 4 Seniors, 1 Business School, 5 Radcliffe.

 

Source: Harvard University. Reports of the President and Treasurer of Harvard College, 1931-32, p. 72.

 

HARVARD UNIVERSITY
1931-32
ECONOMICS 11
Mid-year Examination

Arrange your answers in the order of the questions.
One of the first six questions may be omitted.

  1. “The Classical Economists appreciated the necessity of a fund to support labour during the period of production; but they overlooked the continuous character of production and output, and confused the working capital, which is provided by continuously feeding the flow of available income back into the machine of process, with the liquid capital (goods in stock) at the commencement of any period of process. [Liquid capital is elsewhere defined as “goods yielding nothing, but capable of being used or consumed at any time”; it does not include goods in the hands of merchants.] They did not clearly perceive that the capital to keep labour in employment is found, not in the stocks of goods already available, nor by the abstention from the consumption of available income, but by decisions which have the effect (a) of determining what proportions of the goods emerging from the machine of process are in fixed and in liquid form respectively, and (b) of applying the flow of available income in one way instead of in another, namely, by supporting productive consumers instead of unproductive consumers.” M. Keynes.
    Does the error here described appear in the Classical Economists? and is the criticism of their treatment of abstention valid?
  2. “Marshall’s treatment [of supply] is highly elliptical. A striking illustration of his tendency to telescope his argument is his common practice in his graphs of labelling cost curves and supply curves alike with the symbols s-s’, conventionally used for supply curves, and thus diverting the attention of his readers , and perhaps also occasionally his own attention, from the necessity of selecting from the many possible types of cost curve that one which in the given circumstances alone has claims to being considered as also a supply curve.” Is Marshall open to this criticism? Illustrate and comment.
  3. The bearing (if any) of the concept of a representative firm on the theory of value, of rent, of business profits.
  4. Explain the method by which one can derive the supply price of a commodity produced under conditions of joint supply; that by which one can derive the demand price of a commodity demanded under the conditions of joint demand.
    What bearing, if any, have these methods of analysis on the phenomena of value and distribution in a society which is economically stratified?
  5. “When the artisan or professional man has once obtained the skill required for his work, a part of his earnings are for the future really a quasi-rent of the capital and labour invested in fitting him for his work, in obtaining his start in life, his business connections, and generally his opportunity for turning his faculties to good account; and only the remainder of his income is true earnings of effort. But this remainder is generally a large part of the whole. And here lies the contrast. For when a similar analysis is made of the profits of the business man, the proportions are found to be different: in his case the greater part is quasi-rent.”
    Is the greater part of the earnings of business men to be regarded as quasi-rent? Is the remainder only to be regarded as true earnings of effort?
  6. “The extra income derived from rare natural abilities bears a closer analogy to the surplus produce from the holding of a settler who has made an exceptionally lucky selection, than to the rent of land in an old country.” Is this extra income in the nature of a quasi-rent, in either case?

Not to be omitted.

  1. The following have been suggested, by one writer or another, as the grounds on which the distinction between interest and rent turns:
    1. Land is fixed in amount, instruments made by man are not.
    2. Land is an instrument made by man in essentially the same sense as is any other kind of capital-good; its industrial serviceability and its availability are the result of man’s action.
    3. Competition equalizes the returns on instruments but not those on land.
    4. The returns on land and instruments alike depend on marginal productivity.

Give your own views (briefly) on each point; and sum up with a statement of your conclusion on the tenability of the distinction.

 

HARVARD UNIVERSITY
1931-32
ECONOMICS 11
Final Examination

Arrange your answers in order of the questions.

  1. “With regard to utility, two views are commonly held. The older and more naïve is that an increment of supply (which should always be a continuous stream and not a stock) makes its specific addition to the utility of the total, without affecting the utility of the earlier increments. This is the basis for the familiar utility curve with the implication of consumer’s surplus. On the other hand, it may be held that the utility of all increments is always alike, the addition of each increment to the total bringing down the utility of the earlier ones to the level of its own. Both these views lead to nonsensical results: the first to fantastic magnitudes for total utilities, and the second to the conclusions that the utility of a larger supply may be less than that of a smaller and consequently that people often choose and pay for a reduction in utility.”
    Do these nonsensical results necessarily follow?
  2. “Pure profits are at once necessary and probably non-existent.” What is meant by “pure profits” in this statement? Given the meaning, what do you say to it?
  3. What is the influence of technological improvements on the rate of interest? what the influence of the rate of interest on technological improvements?
  4. “It is obvious that an increase in the supply of capital instruments will make for an increase in the national dividend as a whole. Can it at the same time make for a decrease in the real income of labour? The analysis relevant to this question has been developed by Marshall…. This analysis shows, first, that every factor of production, including entrepreneurs’ work, tends to be remunerated at a rate equivalent to its marginal net product of commodities in general. It shows, secondly, that, other things being equal, the marginal net product, in this sense, of every factor diminishes as the supply of the factor increases beyond a fairly low minimum. This proposition expresses what may be called the law of diminishing returns to individual factors of production. This law must not be confused with the law of diminishing returns to resources in general invested in a given occupation….”
    How far was this analysis developed by Marshall? Are the two laws not to be confused?
  5. Does an elastic demand for one commodity necessarily imply that the demand for some other commodity is inelastic?
  6. What grounds are there for the statement that in Great Britain the elasticity of the aggregate demand for labor is immensely greater than unity?

____________________________________

 1932-33

Course Enrollment: Economics 11
1932-33

[Economics] 11. Professor Taussig.—Economic Theory

Total 42: 33 Graduates, 1 Junior, 6 Radcliffe, 2 Others.

 

Source: Harvard University. Reports of the President and Treasurer of Harvard College, 1932-33, p. 66.

 

HARVARD UNIVERSITY
1932-33
ECONOMICS 11
Mid-year Examination

  1. The original and indestructible powers of the soil; what part they play in Ricardo’s treatment of rent, what in Marshall’s.
  2. “If, for simplicity of exposition, we leave out of account raw materials, the stream of floating capital is constituted almost entirely of wage-goods — goods that are paid over (through money) as wages. Thus, the larger the addition to the normal stream of floating capital that business men can secure in response to a given rise in their interest offer, due to a given improvement in their expectations, the larger proportionately will be the addition made to the real demand for labour. . . .
    “When a boom comes, a large part of the impact is always likely to be upon industries engaged in instrumental trades: and, plainly, extra work there will not lead to an addition to the flow of wage goods — floating capital — for a considerable time. Some part of the primary effect will, however, touch the industries that make these goods and, so far as it does this, we shall have an extra flow of them available to pay for extra labour. This was the important point that the doctrine of the Wages Fund ignored. It must be noticed, however, that this source of additions to floating capital (i.e. extra work) is only available, roughly speaking, so long as unemployed workers are available to be called into industry. If expectations and the desire to employ workpeople go on expanding after this point has been passed, the source is no longer available, and, consequently, the element of elasticity which it accords to the supply of floating capital no longer exists.”
    Was “the important point” here noted in conflict with the Wages Fund doctrine? and is the statement otherwise in conflict with that doctrine?
  3. The tendency of profits to a minimum; how treated by Ricardo, by Mill, by Cairnes?
  4. Explain, with the utmost brevity and precision,

“real cost” of production,
expenses of production,
supply price,
marginal cost,
bulk line cost.

  1. “It may be conceded that if a certain class of people were marked out from their birth as having special gifts for some particular occupation, and for no other, so that they would be sure to seek out that occupation in any case, then the earnings which such men would get might be left out of account as exceptional, when we are considering the chances of success or failure for ordinary persons.”
    Consider whether, given the premise, the conclusion here stated would follow; what is the bearing of the reasoning on Walker’s theory of business profits; what Marshall would say of premise and conclusion.
  2. What bearing, if any, on the concept of non-competing groups do you find on a consideration of, —
    1. universal education, general and technical;
    2. the influence of conventional necessaries;
    3. the representative firm;
    4. the law of derived demand for a commodity demanded jointly with other commodities.

 

HARVARD UNIVERSITY
1932-33
ECONOMICS 11
Final Examination

  1. “Ricardo appears to have seen distinctly almost everything of primary importance in the scientific doctrine of capital, very much as it is known now.” Marshall.
    If so, wherein? If not, wherein not?
  2. — The price of wheat raised on good land is the same as that of wheat raised on the marginal zone, and it affords a surplus above wages and interest paid by farmers for labor and capital used in the tilling of the good land.
    — The existence of this surplus in its original form, that of wheat, affects the supply and the price of that product.
    — The price of cloth woven on good looms is the same as that of equally good cloth woven on marginal ones, and it affords a net surplus above the cost of maintaining the stock of looms and the wages and interest paid by manufacturers for further capital used in connection with the good looms.
    — The existence of this surplus in its original form, that of cloth, affects the supply and the price of this product.
    Discuss (1) the bearing of these statements on the older distinction between capital and land, and (2) the connection between these surpluses and price.
  3. “The diminishing return which arises from an ill proportioned application of the various agents of production into a particular task has little in common with the broad tendency to the pressure of a crowded and growing population on the means of subsistence. . . . It has no very close connection with the tendency of agriculture in an old country to yield a diminishing return to a general increase of resources well applied in cultivation: and indeed exactly parallel cases can be found of a diminishing return to particular resources when applied in undue proportion, even in industries which yield an increasing return to increased applications of capital and labour when appropriately distributed.”
    Is this statement in accord with the general current of economic theory at the present time? Do you agree with it?
  4. “An increase in the supply of capital . . . will make for an increase in the national dividend as a whole. Can it at the same time make for a decrease in the real income of labour? The analysis relevant to this question has been developed by Marshall. Subject to certain important qualifications, which do not affect the present argument, this analysis shows, first, that every factor of production, including entrepreneurs’ work, tends to be remunerated at a rate equivalent to its marginal net product of commodities in general. It shows, secondly, that, other things being equal, the marginal net product, in this sense, of every factor diminishes as the supply of the factor increases beyond a fairly low minimum. . . . This proposition expresses what may be called the law of diminishing returns to individual factors of production. This law must not be confused with the law of diminishing returns to resources in general invested in a given occupation.”
    Wherein does this distinction differ from that contained in the preceding extract? Do you agree with it?
  5. Consider whether it is (1) justifiable, (2) practicable to “charge what the traffic will bear”
    1. when there is a large element of overhead costs;
    2. when there is a large element of joint cost;
    3. when there is simply monopoly;
    4. when there is discriminating monopoly.

____________________________________

1933-34

Course Enrollment: Economics 11
1933-34

 

[Economics] 11. Professor Taussig.—Economic Theory

Total 20: 11 Graduates, 2 Seniors, 5 Radcliffe, 2 Business School.

 

Source: Harvard University. Reports of the President and Treasurer of Harvard College, 1933-34, p. 85.

 

HARVARD UNIVERSITY
1933-34
ECONOMICS 11
Mid-year Examination

One question may be omitted.

  1. “The foundations of the theory [of cost of production and value] as they were left by Ricardo remain intact.” Does Marshall’s treatment of the relation of “general wages” to value bear out this statement? of differences of wages?
  2. Explain
    1. Internal economies of large-scale production.
    2. External economies of large output.
    3. External dis-economies of large output.
  3. “Ricardo, and the economists of his time generally were too hasty in deducing this inference [tendency to increased pressure] from the law of diminishing return; and they did not allow enough for the increase of strength that comes from organization. But in fact every farmer is aided by the presence of neighbours, whether agriculturists or townspeople. . . If the neighbouring market town expands into a large industrial centre, all his produce is worth more; some things which he used to throw away fetch a good price. He finds new openings in dairy farming and market gardening, and with a larger range of produce he makes use of rotations that keep his land always active without denuding it of any of the elements that are necessary for its fertility.” Do you agree?
  4. “The flow of investment of resources for future needs consists of two streams. The smaller consists of new additions to the accumulated stock: the larger merely replaces that which is destroyed; . . . The annual flow of this second stream is probably not less than a quarter of the total stock of capital, even in a country in which the prevailing forms of capital are as durable as in England. It is therefore not unreasonable to assume for the present that the owners of capital in general have been able in the main to adapt its forms to the normal conditions of the time, so as to derive as good a net income from their investments in one way or another.” Has this any bearing on the doctrine of quasi-rent?
  5. If the values of goods were proportional to their real costs, would the utility curve and the demand curve be the same, for persons receiving labor incomes?
  6. What is to be said
    1. of the necessaries of life, as regards elasticity of demand, consumer’s surplus, value and differences of wages;
    2. of conventional necessaries, in the same particulars?
  7. — “The price of wheat raised on good land is the same as that of wheat raised on the marginal zone, and it affords a surplus above wages and interest paid by farmers for labor and capital used in the tilling of the good land.
    — “The fact that farmers pay landlords for this surplus has no effect on the price of wheat.”
    — “The price of cloth woven on good looms is the same as that of equally good cloth woven on marginal ones, and it affords a net surplus above the cost of maintaining the stock of looms and the wages and interest paid by manufacturers for further capital used in connection with the good looms.
    — “The fact that entrepreneurs pay capitalists for this surplus has no effect on the price of cloth.”

What bearing have these passages on the theory of rent? of business profits?

 

HARVARD UNIVERSITY
1933-34
ECONOMICS 11
Final Examination

Arrange your answers in the order of the questions.

  1. Is interest treated as a derivative from “profits”

by Ricardo,
by Marshall,
by Böhm-Bawerk,
by those writers who regard profits as appearing only in a “dynamic” state?

Your own view?

  1. “There is always an interval between the setting to work of a man and the emergence, in consequence of his work, of any finished product, whether for consumption or as a productive instrument for the machine of industry. . . . What is essential is the time interval between the centre of gravity of the labour employed and the output (or, more strictly, the sale) of the finished product. I shall call this interval the period of production.”
    Wherein is the period of production here considered like, and wherein unlike, that discussed by F. A. Walker? by Böhm-Bawerk? For what purposes of economic analysis is the period described in the extract appropriate?
  2. “Autonomous” and “induced” inventions: their bearing on “increasing returns” and on the marginal productivity theorem.
  3. Reflections suggested by a Rembrandt, as regards
    1. market price;
    2. total utility and consumers’ surplus;
    3. the distinction between “wealth” and “capital.”
  4. The problems and distinctions implied in the terms

Economic Welfare,
National Dividend,
Marginal Social Net Product.

____________________________________

 1934-35

Course Enrollment: Economics 11
1934-35

 

[Economics] 11. Professors Taussig and Schumpeter.—Economic Theory

Total 27: 21 Graduates, 1 Senior, 5 Radcliffe.

 

Source: Harvard University. Reports of the President and Treasurer of Harvard College, 1934-35, p. 81.

 

 

Reading List for Economics 11, Fall Semester 1934

Posted from Wolfgang Stolper’s course notes.

HARVARD UNIVERSITY
1934-35
ECONOMICS 11
Mid-year Examination

One question may be omitted. Arrange your answers in the order of the questions.

  1. “Suppose that society is divided into a number of horizontal grades, each of which is recruited from the children of its own members; and each of which has its own standard of comfort, and increases in numbers rapidly when the earnings to be got in it rise above, and shrinks rapidly when they fall below that standard. Suppose, then, that parents can bring up their children to any trade in their own grade, but cannot easily raise them above it and will not consent to sink them below it. . . .”
    Suppose also that there is free competition as regards the earnings of capital.
    On these suppositions what would be the relation between

    1. the values of commodities and their “real cost”;
    2. the values of commodities and their money costs;
    3. the values of commodities and their supply prices?
  2. “Internal economies of large-scale production are primarily a long-run phenomenon, dependent upon appropriate adjustment of scale of plant to each successive output. They should not be confused with the economies resulting from ‘spreading of overhead.’” Why or why not to be thus confused?
    “Internal economies of large-scale production are independent of the size of output of the industry as a whole, and may be accruing to a particular concern whose output is increasing at the same time that the output of the industry as a whole is undergoing a decline.” Why or why not?
  3. Does quasi-rent have the same meaning in the following passages?
    1. “The quasi-rent of farm buildings.”
    2. “When the artisan or professional man has once obtained the skill required for his work, a part of his earnings are for the future really a quasi-rent of the capital and labour invested in fitting him for his work, in obtaining his start in life, his business connections, and generally his opportunity for turning his faculties to good account; and only the remainder of his income is true earnings of effort. But this remainder is generally a large part of the whole. And here lies the contrast. For when a similar analysis is made of the profits of the business man, the proportions are found to be different: in his case the greater part is quasi-rent.”
    3. “In relation to normal value the earnings of high ability are to be regarded as a quasi-rent rather than as a rent proper.”
  4. It is fatal to the conception of consumers’ surplus to admit:
    1. that differences in income make it impossible to measure satisfactions;
    2. that each unit of a homogeneous supply yields ipso facto the same satisfaction as every other unit;
    3. that the satisfaction indicated by the high price paid for an article having “prestige value” will disappear when the article becomes cheap.
  5. Does “capital,” as distinguished from “capital goods,” serve to synchronize the effort of labor with the reward for labor? If so, how? If not, why not?
  6. Explain the distinctions
    1. between the intensive and the extensive margins of cultivation for land;
    2. the intensive and the extensive zones of indifference in the application of labor;
    3. the marginal product of labor and the product of marginal labor.

State summarily your opinion of the usefulness of the distinctions as tools of analysis.

 

Course outline and final exam for Economics 11, Spring Semester 1935

Transcribed from Joseph Schumpeter’s papers and posted earlier.

Source for examination questions: Harvard University Archives. Prof. F. W. Taussig, Examination Papers in Economics 1882-1935 (Scrapbook).

Image Source: Frank W. Taussig in Harvard Class Album, 1934.

Categories
Exam Questions Harvard Suggested Reading Syllabus

Harvard. Graduate Core Economic Theory, Readings and Exams. Schumpeter, 1936-37

 

The reading lists and exams for Schumpeter’s graduate economic theory course in 1935-36 have been posted earlier (the year Paul Samuelson took the course). It is worth noting that Keynes and the General Theory (at least Chapters 11, 13-16) were added to the readings for the second term of 1936-37.

__________________________

Course Announcement, 1936-37

Economics 101 (formerly 11). Economic Theory

Mon., Wed., Fri., at 2. Professor Schumpeter.

Source:  Announcement of the Courses of Instruction Offered by the Faculty of Arts and Sciences during 1936-37 (first edition). Official Register of Harvard University, Vol. XXXIII, No. 5 (March 2, 1936), p. 142.

__________________________

Course Enrollment, 1936-37

Primarily for Graduates:

[Economics] 101. (formerly 11). Professor Schumpeter.–Economic Theory.

Total 36:  30 Graduates, 3 Seniors, 3 Radcliffe.

Source: Harvard University. Report of the President of Harvard College for 1936-1937, p. 93.

__________________________

Economics 101

Following is a list of some of the most important works in English dealing with problems outside the range of perfect competition. They are not all assigned, but assigned reading is taken altogether from this list.

Pigou, A. C., Economics of Welfare, 3rd Edition.
Chamberlin, E. H., The Theory of Monopolistic Competition.
Chamberlin, E. H., On Imperfect Competition, in the March, 1934 Supplement of The American Economic Review, pp. 23-27.
Robinson, Joan, Economics of Imperfect Competition.
Robinson, Joan, What is Perfect Competition, Q. J. E., Nov. 1934.
Zeuthen, F., Problems of Monopoly and Economic Warfare.
Cournot, A. A., Mathematical Principles of the Theory of Wealth.
Edgeworth, F. Y., The Pure Theory of Monopoly (Papers, Vol. I)
Hotelling, Harold, Stability in Competition, E. J., March 1929.
Shove, G. F., The Imperfection of the Market, E. J., March 1933.
Harrod, R. F., Doctrines of Imperfect Competition, Q. J. E., May 1934.
Hicks, J. R., The Theory of Monopoly, Econometrica, Jan. 1935.

The subjects, in the order in which they will be taken up, together with the assigned reading, are given below.

I.  The Technique and the Background.

Pigou, Part II, Ch. XIV.
Robinson, Chs. 1, 2.
Chamberlin, Chs. 1, 2.

 

V. Monopolistic Competition

Chamberlin, Chs. 4, 5, 6, 7.
Robinson, Ch. 7. Q.J.E., Nov. ‘34
Shove, E.J., March ’33.
Harrod, Q.J.E., May ’34.

II.  Simple Monopoly.

Pigou, Part II, Ch. XVI.
Robinson, Chs. 3, 4, 5.

VI. Discrimination.

Pigou, Chs. XVII, XVIII (Part II).
Robinson, Chs. 15, 16.

III.  Duopoly and Oligopoly

Pigou, Part II, Ch. XV.
Chamberlin, Ch. 3.

 
IV. Bilateral Monopoly.

Hicks, Sect. 3.

 

Source: Harvard University Archives. Joseph Schumpeter Papers. Lecture Notes. Box 9, Folder “Ec 11, Fall 1936”.

__________________________

[Hand-written notes, neat, presumably to be typed]

  1. On Substitution

Marshall, V., Chs. 4, 8; VI, Ch.1.
Hicks, Theory of Wages, Ch. 6.
Robinson, Imperfect Competition, Ch. 22.
Machlup, “Commonsense of the Elast. of Subst.”, Rev. Econ. Stud., Vol II, No. 3. (on Econ. 1 shelf)

More Advanced

Hicks, Appendix.
Various notes on elast. of subs. In Vols I and II, Rev. Econ. Stud., by Hicks, Lerner, Kahn, Tarshis etc.
Hicks, Rev. Econ. Stud. Oct., 1936.
Pigou, Econ. Journal, June, 1934.

  1. On Period of Production

Böhm-Bawerk, E., Positive Theory of Capital, Bk II, Ch. 2, 3.
Knight, F. H., “Capital, Time + the Interest Rate,” Economica, August 1934 (on Econ. 151 shelf)
Hayek, F. A., Q. J. E., Feb., ‘36
Machlup, F. “Professor Knight + the Period of Production,” J. P. E., Oct. 1935.

More Advanced

Gifford, C.H.P., Econometrica, April 1935 (in Econ. 102 shelf).

Source: Harvard University Archives. Joseph Schumpeter Papers. Lecture Notes. Box 9, Folder “Ec 11, Fall 1936”.

__________________________

1936-37
HARVARD UNIVERSITY
ECONOMICS 101

(Answer any FIVE questions)

  1. What meaning can be attributed to
    1. Positively inclined demand curves?
    2. Negatively inclined long-run average cost curves?
  2. Define arc elasticity of demand and explain the usefulness or otherwise of the concept.
  3. From given demand curves for consumers’ goods we derive demand curves for the producers’ goods or factors of production. From these in turn we derive the prices of factors and hence incomes. And these incomes determine the demand curves for consumers’ goods. Does this involve circular reasoning?
  4. Why is the explanation of market price by means of the theory of marginal utility superior to the explanation of market price by means of the Ricardian theory of quantity of labor?
  5. Consider a commodity A which is the product of two factors of production B and C. Then “an increase in the supply of A raises the demand for B in terms of money if the elasticity of the demand for A is greater than the elasticity of substitution.” Prove.
  6. Show why and in what sense price is determinate in the case of bilateral monopoly.
  7. “Perfect competition exists to such a negligible extent in the modern economy that all theorizing based on this assumption must be regarded as sheer waste of time.” What have you to say to this?
  8. “The key to problems of imperfect competition lies in the conditions of demand. But it is precisely when we come to problems of imperfect competition that the ordinary demand curve apparatus ceases to have any clear meaning.” Comment.

Mid-Year. 1937.

 

Source: Harvard University Archives. Joseph Schumpeter Papers. Lecture Notes. Box 9, Folder “Ec 11, Fall 1936”.

__________________________

ECONOMICS 101 [“37” is handwritten here]

The first month of the second term will be devoted to a study of the principles underlying the theory of distribution, with special emphasis on wages.

  1. Substitution and Relative Shares
    1. Hicks, J. R., The Theory of Wages, Ch. VI.
    2. Machlup, Fritz, “The Common Sense of the Elasticity of Substitution”, Review of Economic Studies, June, 1935.
    3. Hicks, J. R., “Distribution and Economic Progress: A Revised Version”, Review of Economic Studies, October, 1936.
    4. Also notes and articles on substitution and relative shares in Review of Economic Studies, Vol. I, Nos. 1 and 2, though not required reading, may be consulted.
  2. Theory of Wages and Marginal Productivity
    1. Marshall, Bk. VI, especially Ch. I.
    2. Hicks, J. R., Theory of Wages, Ch. I.
    3. ——-, Marginal Productivity and the Principle of Variation,” Economica, February, 1932.
    4. Schultz, Henry and Hicks, J. R., “Marginal Productivity and the Lausanne School: “A Reply” and “A Rejoinder”, Economica, August, 1932.
    5. Robertson, D. H., “Wage Grumbles” in the volume of essays entitled Economic Fragments.
    6. Chamberlin, E. H., On distribution under Imperfect Competition, pp. 23-27 of the Supplement to the American Economic Review, March, 1934.

Source: Harvard University Archives. Joseph Schumpeter Papers. Lecture Notes. Box 10, Folder “Ec 11, Spring 1937”.

__________________________

 ECONOMICS 101

            The next two or three weeks will be devoted to the discussion of capital and interest. A select bibliography and the assigned reading are listed below.

BIBLIOGRAPHY

  1. Böhm-Bawerk, E., Capital and Interest (a history of interest theories) [: and] The Positive Theory of Capital (the third edition, available only in German, containing the polemical Excursi, is to be preferred to the English translation)
  2. Marx, Karl, Capital (especially Vol. I, Parts III and VII; Vol. II, Part III; Vol. III, Parts II and III)
  3. Wicksell, Knut, Über Wert, Kapital und Rente [, and] Lectures on Political Economy, Vol. I
  4. Fisher, Irving, The Rate of Interest (1907) [; and] The Theory of Interest (1930) (a rewriting of the earlier work)
  5. Taussig, F.W., Wages and Capital
  6. Knight, F.H., “Interest”, article in The Encyc. of Soc. Science
  7. For a rather complete list of the numerous recent articles on capital, interest and the structure of production, Cf. Machlup, Fritz, “Professor Knight and the Period of Production”, Journal of Political Economy, 1935, first footnote.
  8. For an exposition of Böhm-Bawerk, Wicksell and the later work along the same lines done in Sweden, particularly by Gustav Akerman, Cf. Kirchmann, Hans, Studien zur Grenzproduktivitätstheorie des Kapitalzinses.
  9. Keynes, J. M., General Theory of Employment, Interest, and Money.

ASSIGNED READING

  1. Fisher, The Rate of Interest, Part I, Chs. 1,2,3; Part III, Ch. 10
  2. Böhm-Bawerk, Positive Theory, Book I, Ch. 2; Book II, Chs. 2,4,5; Book V, Chs. 1,2,3,4,5; Book VI, Chs. 5,6,7; Book VII, Chs. 1,2,3.
  3. Wicksell, Lectures, Vol. I, pp. 144-171; 185-195.
  4. Keynes, J. M., General Theory of Employment, Interest, and Money, 11, 13, 14, 15, 16.

Source: Harvard University Archives. Joseph Schumpeter Papers. Lecture Notes. Box 10, Folder “Ec 11, Spring 1937”.

__________________________

1936-37
HARVARD UNIVERSITY
ECONOMICS 101

Answer FIVE questions. Arrange your answers in the order of the questions.

  1. Saving, by increasing the quantity of capital, will tend to increase its absolute and relative share. At the same time saving will tend to reduce the rate of interest and thereby to decrease capital’s absolute and relative share. State the conditions on which the net effect of saving on the absolute and relative share will depend. What do you think the actual effect is in practice?
  2. Classical economists spoke of a net benefit accruing from free trade. Have we any means to measure that benefit and to determine how it is divided between the trading nations?
  3. Which of the theories of interest which you have studied seems to you most acceptable and why?
  4. What warrant is there for the statement that in perfect competition and perfect equilibrium every firm will produce that quantity which corresponds to the point of minimum average cost?
  5. Discuss the problem of inequality of incomes from the following points of view: (a) measurement, (b) economic effects, (c) relation to welfare.
  6. Could unemployment exist with perfect competition?
  7. What do you regard as the most desirable railroad rate policy? State clearly and justify your criteria of desirability, and show how the policy selected meets these criteria.

Final. 1937.

 

Source: Harvard University Archives. Joseph Schumpeter Papers. Lecture Notes. Box 10, Folder “Ec 11, Spring 1937”.

Image Source: Harvard University Archives. HUGBS 276.90p(43) Irving Fisher and J. A. Schumpeter (May 12, 1934).